Linear or Non-Linear Differential Equations

Click For Summary
The differential equation (d^4x)/(dt^4) + (1/(1+t))*(d^2x)/(dt^2) = x(t) is classified as linear. It can be expressed in the standard linear form with functions a_k(t) and f(t) without involving any powers of the dependent variable x or its derivatives. The equation features variable coefficients, specifically (1/(1+t)), but remains linear due to the absence of non-linear terms. A key distinction is that if the first derivative were raised to a power, such as (dx/dt)^4, the equation would then be non-linear. Understanding these definitions is crucial for differentiating between linear and non-linear differential equations.
msell2
Messages
13
Reaction score
0
(d4x)/(dt4) + (1/(1+t))*(d2)/(dt2) = x(t)
Is this differential equation linear or non-linear? I don't understand the difference.
 
Physics news on Phys.org
msell2 said:
(d4x)/(dt4) + (1/(1+t))*(d2)/(dt2) = x(t)
Is this differential equation linear or non-linear? I don't understand the difference.

<br /> \frac{\mathrm{d}^4x}{\mathrm{d}t^4} + \frac1{1+t} \frac{\mathrm{d}^2x}{\mathrm{d}t^2} = x<br />
is linear, because it can be written in the form
<br /> a_0(t) x + a_1(t) \frac{\mathrm{d}x}{\mathrm{d}t} <br /> + a_2(t) \frac{\mathrm{d}^2x}{\mathrm{d}t^2} + \dots <br /> + a_n(t) \frac{\mathrm{d}^nx}{\mathrm{d}t^n} = f(t)<br />
for given functions a_k(t) and f(t). It does not, however, have constant coefficients.
 
A differential equation is "linear" as long as there are no functions of the dependent variable, here x, or its derivatives, other than just the usual "linear" functions, multiply or divide by a number and add or subtract.

In particular, that d^4x/dt^4 is just the fourth derivative. Had it been (dx/dt)^4, the first derivative to the fourth power, then the equation would have been non-linear.
 
Question: A clock's minute hand has length 4 and its hour hand has length 3. What is the distance between the tips at the moment when it is increasing most rapidly?(Putnam Exam Question) Answer: Making assumption that both the hands moves at constant angular velocities, the answer is ## \sqrt{7} .## But don't you think this assumption is somewhat doubtful and wrong?

Similar threads

Replies
1
Views
2K
Replies
2
Views
1K
Replies
19
Views
1K
  • · Replies 1 ·
Replies
1
Views
1K
  • · Replies 6 ·
Replies
6
Views
2K
Replies
1
Views
1K
  • · Replies 1 ·
Replies
1
Views
2K
  • · Replies 3 ·
Replies
3
Views
2K
  • · Replies 7 ·
Replies
7
Views
2K
  • · Replies 1 ·
Replies
1
Views
2K